Last visit was: 23 May 2024, 03:08 It is currently 23 May 2024, 03:08
Close
GMAT Club Daily Prep
Thank you for using the timer - this advanced tool can estimate your performance and suggest more practice questions. We have subscribed you to Daily Prep Questions via email.

Customized
for You

we will pick new questions that match your level based on your Timer History

Track
Your Progress

every week, we’ll send you an estimated GMAT score based on your performance

Practice
Pays

we will pick new questions that match your level based on your Timer History
Not interested in getting valuable practice questions and articles delivered to your email? No problem, unsubscribe here.
Close
Request Expert Reply
Confirm Cancel
User avatar
Intern
Intern
Joined: 11 Nov 2004
Posts: 36
Own Kudos [?]: 8 [0]
Given Kudos: 0
Send PM
User avatar
Intern
Intern
Joined: 11 Nov 2004
Posts: 36
Own Kudos [?]: 8 [0]
Given Kudos: 0
Send PM
User avatar
Manager
Manager
Joined: 02 Dec 2004
Posts: 98
Own Kudos [?]: 48 [0]
Given Kudos: 0
Send PM
User avatar
Manager
Manager
Joined: 02 Dec 2004
Posts: 98
Own Kudos [?]: 48 [0]
Given Kudos: 0
Send PM
Re: A recent survey found that more computers than copies of [#permalink]
patrickpui wrote:
this one is tricky. even reading question take more than 2 minute.

I picked C. \

my rationale is that if computers were purchased outside of Germnay but counted in the survey, then it is possible that these overseas purchase might include softwares. Yet since only the computer are counted for sure, these buyers of overseas computers might have purchased legal software that are not counted.

E actually strengthen the argument. people are allowed to make legal copy but these copies are counted as purchase. Then the actual purchase is actually lower than the stated number--more illegal copying is going on.


by the way, even C is somewhat of a weak answer. Nonetheless, this is an except question and whatever is the exception is the key.
User avatar
Intern
Intern
Joined: 11 Nov 2004
Posts: 36
Own Kudos [?]: 8 [0]
Given Kudos: 0
Send PM
Re: A recent survey found that more computers than copies of [#permalink]
I think E counters the argument since it says that germans legally make duplicates of commercial computer programs.How can you assume that the actual purchase is actually lower than the stated number?

I don´t know how to attack these kinds of questions,I think assuming too much is not the right approach.Any comments?



Archived Topic
Hi there,
This topic has been closed and archived due to inactivity or violation of community quality standards. No more replies are possible here.
Where to now? Join ongoing discussions on thousands of quality questions in our Critical Reasoning (CR) Forum
Still interested in this question? Check out the "Best Topics" block above for a better discussion on this exact question, as well as several more related questions.
Thank you for understanding, and happy exploring!
GMAT Club Bot
Re: A recent survey found that more computers than copies of [#permalink]
Moderators:
GMAT Club Verbal Expert
6936 posts
GMAT Club Verbal Expert
238 posts